Đến nội dung

quanghung86 nội dung

Có 489 mục bởi quanghung86 (Tìm giới hạn từ 29-04-2020)



Sắp theo                Sắp xếp  

#668303 VMF's Marathon Hình học Olympic

Đã gửi bởi quanghung86 on 14-01-2017 - 20:24 trong Hình học

Cám ơn Bảo, quá nhanh và nguy hiểm :D, thầy đề nghị bài tiếp.

 

Bài toán 122. Cho tam giác $ABC$, trực tâm $H$, tâm ngoại tiếp $O$. Đường thẳng qua $O$ vuông góc $OA$ cắt trung trực $AH$ tại $D$. $M$ là hình chiếu của $H$ lên $AD$. $HM$ lần lượt cắt $CA,AB,OA$ tại $E,F,G$. đường thẳng qua $H$ song song $OD$ cắt $CA,AB,GC,GB$ lần lượt tại $S,T,K,L$. $SB,TC$ lần lượt cắt $CF,BE$ tại $I,J$. $IK,JL$ lần lượt cắt $MC,MB$ tại $U,V$. Chứng minh rằng $SU,TV,GH,BC$ đồng qui.




#668626 VMF's Marathon Hình học Olympic

Đã gửi bởi quanghung86 on 17-01-2017 - 00:35 trong Hình học

Cám ơn các em đã đóng góp.

 

Figure4266.png

 

Đáp án bài 125. Dựng đường tròn $(B)$ tiếp xúc $AC$. Các tiếp tuyến tại $C,A$ của $(B)$ cắt nhau tại $F$. Từ dữ kiện đề bài dễ thấy hai tam giác $\triangle EDA=\triangle BCF$ g.c.g. Từ đó $EA=BF$ mà $BE\parallel AF$ nên tứ giác $BEAF$ có thể là một hình thang cân hoặc hình bình hành. Nhưng $\angle AEB<\angle AED=\angle CBF<\angle EBF$ do đó $BEAF$ là một hình bình hành. Dễ suy ra $BCDE$ là hình bình hành.




#668232 VMF's Marathon Hình học Olympic

Đã gửi bởi quanghung86 on 13-01-2017 - 21:54 trong Hình học

Cám ơn Bảo vì lời giải rất nhanh và đẹp, theo đề nghị của em thầy để xuất tiếp bài sau

 

Bài toán 117. Cho lục giác lồi $ABCDEF$ thỏa mãn $AB=CD=EF$, $BC=DE=FA$ và $\angle A+\angle B=\angle C+\angle D=\angle E+\angle F$. Chứng minh rằng $\angle A=\angle C=\angle E$.

 

Bài toán 114 của bạn NHN có nhiều phát triển thú vị, mọi người hãy cùng quan tâm.




#667969 VMF's Marathon Hình học Olympic

Đã gửi bởi quanghung86 on 11-01-2017 - 10:21 trong Hình học

Cám ơn Hoàng và Khánh, một lời giải khác cho bài toán 107 của thầy Hà ở đây, rất thú vị và ngắn gọn

 

https://www.artofpro...c6t48f6h1366405

 

Thầy giúp Hoàng đề nghị bài toán sau

 

Bài toán 111. Cho tam giác $ABC$ nội tiếp đường tròn $(O)$. Đường tròn tâm nội tiếp $I$. Đường tròn $(K)$ tiếp xúc $CA,AB$ tại $E,F$ và tiếp xúc trong $(O)$. $P$ là trung điểm cung $BC$ chứa $A$ của $(O)$. $PI$ cắt $BC$ tại $D$. $Q$ là trung điểm cung nhỏ $AP$ của $(O)$. $R$ là trung điểm cung nhỏ $EF$ của $(K)$. $M$ là trung điểm $ID$. Chứng minh rằng $MQ\parallel DR$.

 

Figure4301.png




#667848 VMF's Marathon Hình học Olympic

Đã gửi bởi quanghung86 on 10-01-2017 - 02:34 trong Hình học

Cám ơn Khánh đã giải và gửi bài mới, bài toán của thầy Hà quả thật rất thú vị. Mình xin đưa ra lời giải của mình như sau.

 

Ta chú ý rằng có một tính chất quen thuộc từ cấu hình trên là $BP=CQ=r$ là bán kính nội tiếp tam giác $ABC$. Do đó để đẹp hơn ta nên phát biểu như sau

 

Bài toán 107'. Cho tam giác $ABC$ có đường tròn nội tiếp $(I)$ tiếp xúc với $CA,AB$ tại $E,F$. $P,Q$ đối xứng với $I$ lần lượt qua trung điểm $BE,CF$. $PQ$ cắt $EF$ tại $R$. Chứng minh rằng $IR\parallel BC$.

 

Figure4243.png

 

Lời giải. Gọi $IB,IC$ cắt $EF$ tại $M,N$ dễ thấy $M,N$ nằm trên đường tròn đường kính $BC$. Gọi $BK,CL$ là đường cao của tam giác $ABC$. Ta thấy $FQ\parallel CN$ nên $\angle QFM=\angle CNM=\angle CBM=\angle CLM$. Từ đó tứ giác $FMQL$ nội tiếp nên $QM\perp MN$. Tương tự $PN\perp MN$. Từ biến đổi góc trên dễ thấy hai tam giác vuông $MFQ$ và $MBC$ đồng dạng suy ra hai tam giác $MFB$ và $MQC$ đồng dạng. Tương tự hai tam giác $NEC$ và $NPB$ đồng dạng. Ta thu được $\frac{RN}{RM}=\frac{NP}{MQ}=\frac{NP}{BP}.\frac{CQ}{MQ}=\frac{NE}{EC}.\frac{BF}{FM}=\frac{IN}{IM}.\frac{IN}{IM}$, ta chú ý đẳng thức cuối có do các tứ giác $INFB$ và $IEMC$ nội tiếp. Vậy $\frac{RN}{RM}=\frac{IN^2}{IM^2}$ nên $IR$ tiếp xúc $(IMN)$ do đó $\angle RIN=\angle IMN=\angle ICB$. Từ đó $IR\parallel BC$.

 

Bài toán 108 (Mở rộng ý a) bài toán 7 VMO 2017). Cho tam giác $ABC$ nội tiếp đường tròn $(O)$. Một đường tròn $(K)$ đi qua $B,C$. Trung trực $BC$ cắt $(K)$ tại $M,N$. $P$ là điểm thuộc $(K)$. $PM$ cắt $CA,AB$ tại $E,F$. $BE$ cắt $CF$ tại $L$. Chứng minh rằng $AL,PN$ và $BC$ đồng quy.

 

Figure4244.png




#668627 VMF's Marathon Hình học Olympic

Đã gửi bởi quanghung86 on 17-01-2017 - 00:58 trong Hình học

vietdohoangtk7nqd cần đề nghị bài tiếp nhưng vì lâu nên để bạn ấy đề nghị sau, để topic không bị gián đoán mình xin đề nghị bài tiếp

 

Bài toán 126 (AoPS). Cho tam giác $ABC$ có đường tròn nội tiếp $(I)$ tiếp xúc $BC,CA,AB$ tại $D,E,F$. $P$ nằm trên đường thẳng $OI$ của tam giác $ABC$. $X,Y,Z$ đối xứng $P$ qua $IA,IB,IC$. Chứng minh rằng $DX,EY,FZ$ đồng quy.




#668820 VMF's Marathon Hình học Olympic

Đã gửi bởi quanghung86 on 19-01-2017 - 01:44 trong Hình học

Bài toán 134 là một mở rộng của bài chọn đội tuyển Mỹ mình đã post ở đây trong #11 và các lời giải ở #12,#13 và #15.

 

Ta xét một bài toán tổng quát hơn của bài toán 135, có thể coi là bổ đề.

 

Bài toán 135'. Cho tam giác $ABC$ và $P$ nằm trong tam giác. $PA,PB,PC$ cắt $(O)$ tại $D,E,F$. $X,Y,Z$ đối xứng $D,E,F$ qua trung điểm $BC,CA,AB$. Chứng minh rằng $(XYZ)$ đi qua trực tâm $H$ của $ABC$.

 

Giải. Vì $D,X$ đối xứng qua trung điểm $BC$ nên hai tam giác $ABC$ và $ADX$ có chung trung tuyến hay có chung trọng tâm $G$. Từ đó $X$ là ảnh vị tự trung điểm $U$ của $AD$ qua phép vị tự tâm $G$ tỷ số $-2$. Tương tự $Y,Z$ là ảnh vị tự tâm $G$ tỷ số $-2$ của $V,W$ lần lượt là trung điểm của $BE,CF$. Dễ thấy $O,U,V,W$ nằm trên đường tròn đường kính $OP$ nên $(XYZ)$ đi qua ảnh vị tự của $O$ trong qua phép vị tự tâm $G$ tỷ số $-2$ chính là $H$. Ta hoàn tất chứng minh,

 

Trở lại bài toán 135. Ta chỉ cần $P$ di chuyển trên một đường thẳng cố định đi qua $A$. Gọi đường thẳng đó cắt $(O)$ tại $L$ thì đường tròn $(HRS)$ luôn đi qua đối xứng của $L$ qua trung điểm $BC$ cố định nên tâm của $(HRS)$ thuộc một đường thẳng cố định.

 

Mình xin đề nghị bài tiếp.

 

Bài toán 136. Cho tam giác $ABC$ nội tiếp đường tròn $(O)$ đường tròn bàng tiếp góc $A$ là $(J)$ tiếp xúc $BC$ tại $D$. Đường tròn qua $A,B$ tiếp xúc $(J)$ tại $M$. Đường tròn qua $A,C$ tiếp xúc $(J)$ tại $N$. $BM$ cắt $CN$ tại $P$. Chứng minh rằng $\angle PAB=\angle DAC$.




#670523 VMF's Marathon Hình học Olympic

Đã gửi bởi quanghung86 on 31-01-2017 - 00:20 trong Hình học

Cám ơn Hiếu, đúng là phần quan trọng nhất của bài toán này là từ bổ đề 2. Tuy nhiên việc phát biểu bài toán là từ một bài Iran trên AoPS, trong đó cũng có lời giải của Bảo. Em hãy đề nghị tiếp một bài toán cho topic được duy trì.




#670079 VMF's Marathon Hình học Olympic

Đã gửi bởi quanghung86 on 26-01-2017 - 23:39 trong Hình học

Bài toán 148 có thể tham khảo tại đây trong #4 của Telv Cohl.




#670078 VMF's Marathon Hình học Olympic

Đã gửi bởi quanghung86 on 26-01-2017 - 23:29 trong Hình học

Lời giải bài toán 149.

 

Figure4287.png

 

Sử dụng định lý Desargues thì bài toán cần chứng minh tương đương với giao điểm của các cặp đường thẳng $(B_cC_b,BC);(C_aA_c,AC);(A_bB_a,AB)$ thẳng hàng khi và chỉ khi giao điểm của $(A_bA_c,BC);(B_cB_a,AC);(C_aC_b,AB)$ thẳng hàng, Gọi $B_cC_b$ và $A_bA_c$ cắt $BC$ lần lượt tại $A_3,A_4$ thì áp dụng định lý Menelaus ta thấy $\frac{A_3B}{A_3C}=\frac{B_cP}{B_cC}.\frac{C_bB}{C_bP}$ và $\frac{A_4B}{A_4C}=\frac{A_cP}{A_cC}.\frac{A_bB}{A_bP}$ do đó 

 

Tương tự ta có $\frac{B_3C}{B_3A}=\frac{C_aP}{C_aA}.\frac{A_cC}{A_cP}$ và $\frac{B_4C}{B_4A}=\frac{B_aP}{B_aA}.\frac{B_cC}{B_cP}$ và $\frac{C_3A}{C_3B}=\frac{A_bP}{A_bB}.\frac{B_aA}{B_aP}$ và $\frac{C_4A}{C_4B}=\frac{C_bP}{C_bB}.\frac{C_aA}{C_aP}$. 

 

Từ đó $\frac{A_3B}{A_3C}.\frac{B_3C}{B_3A}.\frac{C_3A}{C_3B}=\frac{B_cP}{B_cC}.\frac{C_bB}{C_bP}.\frac{C_aP}{C_aA}.\frac{A_cC}{A_cP}.\frac{A_bP}{A_bB}.\frac{B_aA}{B_aP}$

 

Và $\frac{A_4B}{A_4C}.\frac{B_4C}{B_4A}.\frac{C_4A}{C_4B}=\frac{A_cP}{A_cC}.\frac{A_bB}{A_bP}.\frac{B_aP}{B_aA}.\frac{B_cC}{B_cP}.\frac{C_bP}{C_bB}.\frac{C_aA}{C_aP}$

 

Vậy  $\frac{A_3B}{A_3C}.\frac{B_3C}{B_3A}.\frac{C_3A}{C_3B}=1/(\frac{A_4B}{A_4C}.\frac{B_4C}{B_4A}.\frac{C_4A}{C_4B})$ nên $\frac{A_3B}{A_3C}.\frac{B_3C}{B_3A}.\frac{C_3A}{C_3B}=1\iff \frac{A_3B}{A_3C}.\frac{B_3C}{B_3A}.\frac{C_3A}{C_3B}=1$.

 

Ta hoàn thành chứng minh.

 

Theo cách chứng minh bài này thì chỉ cần $B_a,C_a$ bất kỳ trên đường thẳng $PA$ là được và tương tự các điểm còn lại :D!

 

Bài toán 150 (AoPS). Cho tam giác $ABC$ với $M$, $N$, $Q$ là trung điểm $AB$, $BC$ và $AC$. $P$ nằm trong tam giác và nằm trên phân giác $\angle BCA$. Gọi $D=AP\cap MN$ và $E=BP\cap MQ$. Chứng minh rằng $MD=ME$.




#669146 VMF's Marathon Hình học Olympic

Đã gửi bởi quanghung86 on 20-01-2017 - 23:24 trong Hình học

Cách của thầy cũng giống hệt cách của Tuấn là tính hết ra và kết quả cuối cùng đúng như thế kia. Tuy nhiên Phước có cách giải thuần túy hình khá đẹp các bạn hãy cứ thử sức tiếp. Tuấn đề nghị một bài tiếp đi em!




#666885 VMF's Marathon Hình học Olympic

Đã gửi bởi quanghung86 on 03-01-2017 - 23:30 trong Hình học

Đáp án bài toán 100.

 

Trong suốt bài toán này nếu $X$ là một điểm nằm trên đường tròn ngoại tiếp tam giác $ABC$ thì ta ký hiệu $s_X$ chỉ đường thẳng Simson của $X$ ứng với tam giác $ABC$. Ta sử dụng các bổ đề sau

 

Bổ đề 1. Cho tam giác $ABC$ nội tiếp đường tròn $(O)$ và $M,N$ thuộc $(O)$ thì $(s_M,s_N)=\frac{1}{2}(\overrightarrow{ON},\overrightarrow{OM})(\bmod\pi)$.
 

Bổ đề 2. Cho tam giác $ABC$ nội tiếp đường tròn $(O)$ và trực tâm $H$ và $P$ là một điểm bất kỳ trên $(O)$ thì $s_P$ đi qua trung điểm $PH$.
 

Hai bổ đề là các bài toán cơ bản xin không nêu cách chứng minh.

 

Bổ đề 3. Cho tam giác $ABC$ nội tiếp đường tròn $(O)$ và $M,N$ là hai điểm thuộc $(O)$. $P$ là một điểm bất kỳ trên đường tròn đường kính $MN$. Gọi $m,n$ là hai đường thẳng lần lượt qua $M,N$ theo thứ tự song song với $s_M,s_N$. Khi đó thì đối xứng của $m,n$ lần lượt qua $PM,PN$ cắt nhau trên $(O)$.

 

Figure2231.png

 

Chứng minh. Bổ đề được chứng minh đơn giản bằng cộng góc. Thật vậy, gọi $m'$ và $n'$ lần lượt là đối xứng của $m,n$ qua $PM,PN$. Ta có biến đổi góc

 

$(m',n')=(m',PM)+(PM,PN)+(PN,n')(\bmod\pi)$

 

$=(PM,m)+(PN,PM)+(n,PN)(\bmod\pi)$ (Do $PM\perp PN$ nên $(PM,PN)=\frac{\pi}{2}=-\frac{\pi}{2}=(PN,PM)(\bmod\pi)$)

 

$=(n,m)(\bmod\pi)$

 

$=\frac{1}{2}(\overrightarrow{OM},\overrightarrow{ON})(\bmod\pi)$.

 

Ta chú ý $m'$ và $n'$ đi qua $M,N$ nên từ đó $m'$ và $n'$ cắt nhau trên $(O)$. Ta có điều phải chứng minh.

 

Giải bài toán.

 

Figure2232.png

 

Gọi $K',L'$ là ảnh của $K,L$ qua phép vị tự tâm $H$ tỷ số $2$. Vì $S$ cũng là ảnh của $T$ qua phép vị tự tâm $H$ tỷ số $2$ nên ta chỉ cần chứng minh $S$ là trung điểm của $K'L'$ thì bài toán được giải quyết. Thật vậy, theo bổ đề 2 dễ thấy phép vị tự tâm $H$ biết $s_M,s_N$ thành các đường thẳng $K'M,K'N$. Từ đó ta có $(K'M,K'N)=(s_M,s_N)=\frac{1}{2}(\overrightarrow{ON},\overrightarrow{OM})(\bmod\pi)$.

 

Vậy nếu gọi $R_1$ là đối xứng của $K'$ qua $MN$ thì $(R_1M,R_1N)=-(K'M,K'N)=\frac{1}{2}(\overrightarrow{OM},\overrightarrow{ON})(\bmod\pi)$ vậy suy ra $R_1$ thuộc $(O)$.

 

Tương tự nếu $R_2$ đối xứng $L'$ qua $PQ$ thì $R_2$ thuộc $(O)$.

 

Mặt khác từ việc lấy đối xứng ta có $R_1N,R_2P$ lần lượt là đối xứng của $NK',PL'$ theo thứ tự qua $SN,SP$. $S$ lại nằm trên đường tròn đường kính $NP$ vậy theo bổ đề 3 thì hai đường thẳng này phải có chung một điểm thuộc $(O)$. Vậy từ đó $R_1\equiv R_2\equiv R\in (O)$. Từ đó theo tính chất đối xứng trục, hai trục $MN,PQ$ vuông góc do đó $L'=\mathcal{R}_{PQ}(R)=\mathcal{R}_{PQ}[\mathcal{R}_{MN}(K')]=\mathcal{R}_{PQ}\circ \mathcal{R}_{MN}(K')=\mathcal{S}_{S}(K')$ vậy $K'$ và $L'$ đối xứng nhau qua $S$. Từ các nhận xét ban đầu ta thu được điều phải chứng minh.




#666738 VMF's Marathon Hình học Olympic

Đã gửi bởi quanghung86 on 02-01-2017 - 23:13 trong Hình học

Do sơ suất mình đã đề nghị trùng bài toán 36, cám ơn Hoàng đã cho lời giải mới cho bài đó. Cám ơn Dương đã đóng góp lời giải, mình xin đề xuất lại đề toán 101.

 

Bài toán 101. Cho tam giác $ABC$ với đường cao $AH$. $M,N$ thuộc $BC$ sao cho $AM\perp AB, AN\perp AC$. $I$ là tâm nội tiếp tam giác $AMN$. $NI,MI$ cắt $CA,AB$ lần lượt tại $E,F$. Gọi $K,L$ là tâm nội tiếp tam giác $AHM,AHN$. Chứng minh rằng $EK,FL$ và $IH$ đồng quy.




#641507 VMF's Marathon Hình học Olympic

Đã gửi bởi quanghung86 on 21-06-2016 - 01:56 trong Hình học

Trình bày lại latex cho Huy, lời giải trên đúng tuy nhiên phải dùng tới conic. Sau đây là lời giải dùng nghịch đảo

 

Giải. Chú ý rằng đường tròn $(M)$ qua $A,P$ và tiếp xúc $(PBC)$. Như vậy $(M)$ cắt $CA,AB$ tại $E',F'$ là đối xứng của $A$ qua $E,F$. Tương tự đường tròn $(N)$ qua $A,Q$ và tiếp xúc $(QBC)$ cắt $CA,AB$ tại $G',H'$ là đối xứng của $A$ qua $G,H$. Vậy ta cần chứng minh $E',F',G',H'$ cùng thuộc một đường tròn này tiếp xúc $BC$. Nghịch đảo cực $A$ phương tích bất kỳ. Chú ý rằng hai điểm đẳng giác vẫn có ảnh là hai điểm đẳng giác. Ta thu được bài toán sau của Bảo và Dương đã cm và phát biểu tại đây.

 

Cho tam giác $ABC$ có $P,Q$ là hai điểm đẳng giác. Tiếp tuyến tại $P$ của $(PBC)$ cắt $CA,AB$ tại $E,F$. Tiếp tuyến tại $Q$ của $(QBC)$ cắt $CA,AB$ tại $G,H$. Thì $E,F,G,H$ cùng thuộc một đường tròn tiếp xúc $(O)$.




#643012 VMF's Marathon Hình học Olympic

Đã gửi bởi quanghung86 on 30-06-2016 - 21:15 trong Hình học

Cám ơn Dương đã gửi link, thầy có thể tổng kết lại được là bài tổng quát nên phát biểu như sau sẽ đẹp

 

Cho tam giác $ABC$ với trực tâm $H$ và tâm ngoại tiếp $O$. $P$ nằm trên trung trực $BC$ và $Q$ đẳng giác $P$. $PQ$ cắt $OH$ tại $R$. Chứng minh rằng $\frac{\overline{RH}}{\overline{RO}}=2-\frac{BC^2}{PC^2}$.

 

Lời giải trong link của Telv đã rõ ràng, tuy nhiên khi phát biểu như trên liệu có các đi khác ?

 

Figure3915.png




#640183 VMF's Marathon Hình học Olympic

Đã gửi bởi quanghung86 on 14-06-2016 - 02:06 trong Hình học

Topic hơi gián đoạn do bài toán 35 của Dương khá khó, tạm gác lại bài đó, mình xin đề xuất một bài tiếp như sau

 

$\boxed{\text{Bài toán 36.}}$ Cho tam giác $ABC$ cân tại $A$ nội tiếp đường tròn $(O)$ với $AD$ là đường kính của $(O)$. Các điểm $E,F$ lần lượt thuộc $DC,DB$. $G$ thuộc $EF$ sao cho $\frac{GF}{GE}=\frac{FB}{CE}$. Chứng minh rằng $CG$ và $AF$ cắt nhau trên $(O)$.

 

Khi đề xuất bài mình nghĩ nên có một số thứ tự các tiêu chí ưu tiên như sau

 

1) Đúng

2) Hay, đẹp

3) Mới

4) Sát với thi Olympic

5) Kiến thức đơn giản

6) Đề bài ngắn gọn, dễ hiểu

7) Nhiều hướng tiếp cận, tổng quát.




#636016 VMF's Marathon Hình học Olympic

Đã gửi bởi quanghung86 on 27-05-2016 - 20:20 trong Hình học

Nguồn gốc bài 9 http://www.artofprob...unity/c6h486466

Nguồn gốc bài 10 http://www.artofprob...unity/c6h470341




#635826 VMF's Marathon Hình học Olympic

Đã gửi bởi quanghung86 on 27-05-2016 - 00:18 trong Hình học

Cám ơn Bảo đã post lại bài mở rộng bài toán miền Tây, sau đây là lời giải dùng hoàn toàn ý tưởng của Bảo được thầy làm chi tiết hơn.

 

$\boxed{\text{Lời giải bài toán 6.}}$ Ta dễ thấy các đường tròn $(GBE),(GCF)$ đi qua $K$ nên ta viết lại gọn hơn như sau

 

$\boxed{\text{Bài toán 6'.}}$ Cho tam giác $ABC$ có tâm ngoại tiếp $O$. Một đường tròn $(K)$ qua $B,C$ cắt $CA,AB$ tại $E,F$. $BE$ cắt $CF$ tại $H$. Gọi $M,N$ là tâm ngoại tiếp các tam giác $KBE,KCF$. $FM$ cắt $EN$ tại $L$. Chứng minh rằng $H,L,O$ thẳng hàng. 
 
Ta dùng các bổ đề sau
 
Bổ đề 1. Cho tam giác $ABC$ với $E,F$ bất kỳ trên đường thẳng $CA,AB$ thì trực tâm $H$ của tam giác $ABC$ có cũng phương tích với đường tròn đường kính $BE,CF$.
 
Bổ đề này là kết quả quen thuộc xin không nêu chứng minh.
 
Bổ đề 2. Cho tứ giác $ABCD$ có $AB$ cắt $CD$ tại $E$. $AD$ cắt $BC$ tại $F$. Thì đường tròn đường kính $AC,BD,EF$ đồng trục.
 
Figure3831.png
 
Chứng minh. Gọi $H,K$ là trực tâm tam giác $ECD,FBC$ theo bổ đề trên dễ thấy $H,K$ có cùng phương tích với các đường tròn đường kính $AC,BD,EF$.
 
Bổ đề 3. Cho tam giác $ABC$ có $E,F$ thuộc $CA,AB$ sao cho tứ giác $BCEF$ nội tiếp. $P$ là điểm bất kỳ. Lấy $Q$ sao cho $QE\perp AC,QF\perp AB$. $PC,PB$ cắt $QF,QE$ tại $M,N$. Lấy $R$ sao cho $BR\perp AM,CR\perp AN$. Chứng minh rằng $P,Q,R$ thẳng hàng.
 
Figure3832.png
 
Chứng minh. Gọi $RB,RC$ cắt $AM,AN$ tại $K,L$. $MN$ cắt $BC$ tại $S$. $G$ là hình chiếu của $P$ lên $AS$. Dễ thấy $K,F$ thuộc đường tròn đường kính $BM$ và $L,N$ thuộc đường tròn đường kính $CN$. Nên $AK.AM=AF.AB=AE.AC=AL.AN$. Từ đó dễ thấy $A$ nằm trên trục đẳng phương của các đường tròn đường kính $BM,CN$. Theo bổ đề thì $A$ cũng nằm trên trục đẳng phương chung của các đường tròn đường kính $BM,CN,PS$. Từ đó $AG.AS=AK.AM=AF.AB=AE.AC=AL.AN$. Từ đẳng thức này dễ thấy $G$ nằm trên các đường tròn $(SBF),(SEC)$ suy ra $G$ nằm trên $(AEF)$ là đường tròn đường kính $AQ$ do đó $P,Q,G$ thẳng hàng. Cũng từ đẳng thức thì $G$ nằm trên đường tròn $(ANL),(AMK)$ suy ra $G$ nằm trên $(AKL)$ là đường tròn đường kính $AR$. Từ đó $P,R,G$ thẳng hàng. Như vậy $P,Q,R$ thẳng hàng.
 
Chú ý bổ đề 3 là của Bảo và chứng minh được tham khảo ở link.
 
Hệ quả. Cho tam giác $ABC$ cân tại $A$ với tâm ngoại tiếp $O$. $P$ bất kỳ. $PB,PC$ cắt trung trực $CA,AB$ tại $N,M$. Lấy $R$ sao cho Lấy $R$ sao cho $BR\perp AM,CR\perp AN$. Chứng minh rằng $P,O,R$ thẳng hàng.
 
Figure3833.png
 
Chứng minh. Gọi $E,F$ là trung điểm $CA,AB$ thì tứ giác $BCEF$ nội tiếp. Áp dụng trực tiếp bổ đề 3 trên ta có điều phải chứng minh.
 
Figure3834.png
 
Giải bài toán. Gọi $I$ là tâm, $(KEF)$ theo kết quả này thì $I$ nằm trên $OH$. Dễ thấy $IM,IN$ là trung trực của $KF,KE$. Đến đây áp dụng hệ quả vào tam giác $KEF$ suy ra $L,H,I$ thẳng hàng. Từ đó $HL$ đi qua $O$.
 
Nhận xét. Bài toán này là cách viết khác của bài toán nằm trong chuỗi mở rộng bài toán miền tây Trung Quốc năm 2014 sau đây
 
Bài toán. Cho tam giác $ABC$ có tâm ngoại tiếp $O$. Một đường tròn $(K)$ qua $B,C$ cắt $CA,AB$ tại $E,F$. $BE$ cắt $CF$ tại $H$. Gọi $M,N$ là tâm ngoại tiếp các tam giác $KCE,KBF$. $BM$ cắt $CN$ tại $P$. Chứng minh rằng $AP$ đi qua tâm $(HEF)$.
 
Bài toán này có thể xem tại đây.
 
Figure3835.png
 
Toàn bộ lời giải trên là của Nguyến Đức Bảo trong đó bổ đề 3 là quan trọng nhất, lời giải nghịch đảo trong link được tác giả viết lại không nghịch đảo đơn giản hơn nhưng ý tưởng vẫn vậy. Hệ quả có cách chứng minh trực tiếp ngắn hơn.
 
$\boxed{\text{Bài toán 7.}}$ Cho tam giác $ABC$ có đường cao $AH$ và tâm nội tiếp $I$. $IB,IC$ cắt $AH$ tại $M,N$. $K,L$ là tâm ngoại tiếp tam giác $MAB,NAC$. $R,Q$ là trung điểm $IB,IC$. Lấy $P$ sao cho $PR\perp NL,PQ\perp MK$. Một tiếp tuyến thay đổi của đường tròn Euler của tam giác $IBC$ cắt $PQ,PR$ tại $S,T$. $X$ đối xứng $P$ qua $S,T$. Chứng minh rằng đường tròn $(XST)$ luôn tiếp xúc một đường tròn cố định khi tiếp tuyến thay đổi.
 
Nguồn. Sáng tác + Serbia năm 2016
 
Xin phép cho mình không tham gia ghi điểm, vì thời gian mình không có nhiều để thường xuyên giải. Có thể coi mình là phương án trợ giúp trong các tình huống khó xử lý của topic này. Mình ủng hộ nhiệt tình và rất vui vì topic này ra đời quy tụ nhiều lời giải và bài toán hình học Olympic hay. 



#643321 VMF's Marathon Hình học Olympic

Đã gửi bởi quanghung86 on 02-07-2016 - 19:27 trong Hình học

Nguồn gốc của bài toán 57 là bài số 14 trong này http://geometry.ru/o...13/zaochn-e.pdf

 

Đáp án ở đây http://geometry.ru/o...13/zaochsol.pdf

 

Bài này là bài toán hay có nhiều phát triển và ứng dụng.




#643834 VMF's Marathon Hình học Olympic

Đã gửi bởi quanghung86 on 06-07-2016 - 12:23 trong Hình học

Đây là link gốc bài toán 60' 

 

http://www.artofprob...unity/c6h612592




#649655 VMF's Marathon Hình học Olympic

Đã gửi bởi quanghung86 on 14-08-2016 - 20:05 trong Hình học

Bài toán 88 đã có đáp án trên báo Epsilon 10, mình xin đề nghị một bài mới

 

$\boxed{\text{Bài toán 89.}}$ (Tập huấn đội dự tuyển KHTN 2016) Cho tam giác $ABC$ nhọn có đường cao $AA_0,BB_0,CC_0$, trung tuyến $AA_1,BB_1,CC_1$ tâm đường tròn Euler là $N$. Đường thẳng qua $A$ song song $BC$ cắt $B_0C_0$ tại $A_2$. $A_3$ là hình chiếu của $A_2$ lên $AA_1$. $A_4$ là trung điểm của $AA_2$. $A_5$ là đối xứng của $A_3$ qua $NA_4$. Tương tự có $B_5,C_5$. Chứng minh rằng $AA_5,BB_5,CC_5$ đồng quy.




#646777 VMF's Marathon Hình học Olympic

Đã gửi bởi quanghung86 on 27-07-2016 - 19:35 trong Hình học

Hệ quả đẹp của bài toán 86, hai tam giác $IAC,IBD$ có chung đường đối trung.




#646728 VMF's Marathon Hình học Olympic

Đã gửi bởi quanghung86 on 27-07-2016 - 15:47 trong Hình học

Cám ơn Khánh, đúng là bài này ra như vậy bị dễ đi, ý tưởng là thầy muốn chứng minh $PL$ đi qua điểm Lemoine của $ABC$. Vậy ta tiếp tục

 

$\boxed{\text{Bài toán 86.}}$ Cho tứ giác $ABCD$ ngoại tiếp đường tròn $(I)$. Chứng minh rằng phân giác các góc $\angle AIC,\angle BID$ vuông góc.




#643835 VMF's Marathon Hình học Olympic

Đã gửi bởi quanghung86 on 06-07-2016 - 12:25 trong Hình học

$\boxed{\text{Bài toán 64.}}$ Cho tam giác $ABC$ nội tiếp trong đường tròn $(O)$. $P$ bất kỳ nằm trong tam giác. $PB,PC$ lần lượt cắt $(O)$ tại $E,F$ khác $B,C$. $M,N$ lần lượt là đối xứng của $B,C$ qua $AF,AE$. $K$ là tâm ngoại tiếp tam giác $PMN$. Gọi $L$ đối xứng $K$ qua trung trực $AP$. Chứng minh rằng $PL\parallel AO$ khi và chỉ khi $\frac{PB}{PC}=\frac{AB}{AC}$.




#635264 VMF's Marathon Hình học Olympic

Đã gửi bởi quanghung86 on 24-05-2016 - 20:36 trong Hình học

Các giải khác cho bài 5 (dùng hình của Bảo) tránh dùng sin, chú ý hai tam giác $XBF$ và $XEC$ đồng dạng nên tỷ lệ diện tích là bình phương tỷ lệ đồng dạng.

Gọi $AX$ cắt $BC$ tại $U$ thì $\frac{UB}{UC}=\frac{[XAB]}{[XAC]}=\frac{[XAB]}{[XBF]}.\frac{[XBF]}{[XEC]}.\frac{[XEC]}{[XAC]}=\frac{AB}{BF}.\frac{BF^2}{CE^2}.\frac{CE}{AC}=\frac{AB.BF}{AC.CE}$. Tương tự dùng Ceva ta có điều phải chứng minh.

 

Chú ý hai tam giác $XBF$ và $XEC$ đồng dạng cùng hướng, ta có thể thay bằng diện tích đại số và độ dài đại số.